Search found 1448 matches


Hi, t1 is odd t2 is odd t3 is even t4 = t3+t2 = even+odd = odd t5 = t4+t3 = odd+even = odd t6 = t5+t4 = odd+odd = even t7 = t6+t5 = even+odd = odd t8 = t7+t6 = odd+even = odd t9 = t8+t7 = odd+odd = even . . You can see the pattern, every third element is even and others are odd. So, if there are 300...

by Frankenstein

Sun Aug 21, 2011 6:48 am
Forum: Problem Solving
Topic: Sequence (Fibonacci numbers)
Replies: 1
Views: 1580

Hi Mitch/Frank, whats the takeaway from this problem ? Lame analogy : If a and b complete 1/2 of a given work in 10 days then combined rate = 1/20. If a's rate is 1/80 then we can calculate b's rate = 1/20 - 1/80 = 3/80. - {sufficient} Why does above hypothesis not work at the problem in the thread...

by Frankenstein

Sun Aug 21, 2011 6:40 am
Forum: Data Sufficiency
Topic: rate prb-tricky
Replies: 8
Views: 2551

bblast wrote: Quick question - Do we still call the set-"an evenly spaced set" after removing the other pairs (other than 1,19) ? I hope NO ?
You are right..A big 'NO' :)

by Frankenstein

Sat Aug 20, 2011 10:20 pm
Forum: Data Sufficiency
Topic: Standar Deviation
Replies: 7
Views: 3334

Hi, Mean of the given set is 10. From(1): The two numbers picked can be any of the following pairs: (1,19),(3,17),...(9,11). So, depending on the pair picked, we get different values for standard deviation. Not sufficient From(2): Same Hence, E But i did not understand 2 numbers are selected means? ...

by Frankenstein

Sat Aug 20, 2011 10:07 pm
Forum: Data Sufficiency
Topic: Standar Deviation
Replies: 7
Views: 3334

I didn't understand statement 2. Does it mean that the sum of the n consecutive integers is a multiple of 9? Yes, but not directly. If the n consecutive integers are 2,3,4,5,6,7.. then 234567(6- digit number) is divisible by 9. Anyway for a number to be divisible by 9, the sum of its digits should ...

by Frankenstein

Sat Aug 20, 2011 11:26 am
Forum: Data Sufficiency
Topic: Sum of consecutive integers
Replies: 7
Views: 1117

Hi Adam, IR section has been tested till 19th August(from 5th July to 19th August). As far as I know, GMAC never said it would refund 250$, but it did say it would reward something around 30$(don't remember exactly but definitely not more than 50$) if we put efforts on that section. So, there is no ...

by Frankenstein

Sat Aug 20, 2011 11:21 am
Forum: I just Beat The GMAT!
Topic: Question for recent test-takers from an Expert
Replies: 5
Views: 2299

Condition 2 -- Consider 11+12+13 = 36 -even Consider 2+3+4+5+6+7 = 27 -- ODD... Not sufficient. Hence answer = A Hi, Minor mistake. You cannot choose 11,12,13. If you are choosing this set, you are picking n as 3. When we write those numbers in a series it becomes 111213, which is not 'n' digit num...

by Frankenstein

Sat Aug 20, 2011 11:08 am
Forum: Data Sufficiency
Topic: Sum of consecutive integers
Replies: 7
Views: 1117

Hi knight,
Let V be the volume of the tank.
Time taken is actually V/R1 + V/R2.
So, V/R1 + V/R2 = 30.
IF you are making it 1/R1 + 1/R2, then the assumption is that the volume of the tank is 1 Lt.
That is the reason, we need both statements.

by Frankenstein

Sat Aug 20, 2011 11:04 am
Forum: Data Sufficiency
Topic: rate prb-tricky
Replies: 8
Views: 2551

From (1) A(A²-1)<0 Meaning either A>0 and (A²-1)<0 Or A<0 and (A²-1)>0 This gives us conflicting values of A and hence INSUFFICIENT. Is my reasoning sufficient? Do I need to do any further testing? Hi, I don't clearly understand what you mean by conflicting values. Anyway, my take on this: a>0 a...

by Frankenstein

Sat Aug 20, 2011 5:41 am
Forum: Problem Solving
Topic: Inequalities
Replies: 1
Views: 948

Hi, From(1): If n is even, average of n consecutive number is the average of two middle most terms, which is (even+odd)/2 making it a fraction. It will never be equal to 1. Sufficient From(2): 0<S<n. So, o/n<S/n<n/n So, 0<average<1 So, average is not equal to 1. Sufficient Hence, D

by Frankenstein

Sat Aug 20, 2011 4:11 am
Forum: Data Sufficiency
Topic: consecutive integers
Replies: 2
Views: 1102

Hi,
From(1):
Sum of integers from a to (a+5) = 6a+15. This is always odd.
Sufficient

From(2):
if the number is 45, sum is odd
if the number is 3456, sum is even
Not sufficient

Hence, A

by Frankenstein

Sat Aug 20, 2011 4:01 am
Forum: Data Sufficiency
Topic: Sum of consecutive integers
Replies: 7
Views: 1117

I have one question though. Which is better : more and more littered or littered more and more ? Hi, Meaning is the issue here: more and more littered -> the level of littering is increasing. littered more and more -> littering is done more number of times. Original sentence says: more and more lit...

by Frankenstein

Sat Aug 20, 2011 1:22 am
Forum: Sentence Correction
Topic: Modifier doubt ?
Replies: 8
Views: 2079

Hi, Bag A -> Red : White = 1:3. So, white is a multiple of 3 White : Blue = 2:3. So, white is a multiple of 2 So, white is a multiple of 2 and 3. So, Red:White:Blue = 2:6:3 or 2a:6a:3a Bag B -> Red : White = 1:4 or b:4b 6a+4b = 30 So, only possible positive integral solutions are: a=1,b=6 a=3,b=3 So...

by Frankenstein

Sat Aug 20, 2011 1:12 am
Forum: Problem Solving
Topic: MGMAT abstract probability qn - need expert help !
Replies: 2
Views: 1061

No probs. Yes, there is a scope for improvement of the wording in the question.

by Frankenstein

Sat Aug 20, 2011 12:52 am
Forum: Problem Solving
Topic: Knewton Problem doubt- Experts Help!
Replies: 5
Views: 1181

Totally agree mate. That is the trickiest part indeed, at least for me. Practice and alertness should make us better!

by Frankenstein

Sat Aug 20, 2011 12:50 am
Forum: GMAT Strategy
Topic: RC: Main Idea vs. Detail
Replies: 10
Views: 4526